The function f(x)=8x+3x^−1 has one local minimum and one local maximum.
This function has a local maximum at x= With a value of =
This function has a local minimum at x = With a value of =

Answers

Answer 1

The function f(x) = 8x + 3x^(-1) has a local maximum at x = 2 with a value of f(2) = 19, and a local minimum at x = -2 with a value of f(-2) = -19.Explanation:

To find the local extrema of a function, we need to find the critical points of the function, which are the points where the derivative is either zero or undefined. In this case, the derivative of f(x) is f'(x) = 8 - 3x^(-2), which is undefined at x = 0.Setting the derivative equal to zero, we get:8 - 3x^(-2) = 0Solving for x, we get:x = ±2

These are the critical points of the function. To determine whether each critical point is a local maximum or a local minimum, we need to examine the second derivative of the function.

The second derivative of f(x) is f''(x) = 6x^(-3), which is negative for x > 0 and positive for x < 0.Therefore, x = 2 is a local maximum of the function with a value of f(2) = 19, and x = -2 is a local minimum of the function with a value of f(-2) = -19. These are the only local extrema of the function, since the function is increasing for x < -2 and decreasing for -2 < x < 0, and then increasing again for x > 0.

For more similar questions on topic local maximum

brainly.com/question/7805334

#SPJ11


Related Questions

Consider the vector field.
F(x, y, z) =
5ex sin(y), 9ey sin(z), 2ez
sin(x)
(a) Find the curl of the vector field.
curl F =
(b) Find the divergence of the vector field.
div F =

Answers

(a) To find the curl of the vector field F(x, y, z), we first find its component functions:

F(x, y, z) = (5ex sin(y), 9ey sin(z), 2ez sin(x))

Then, we use the formula for the curl of a vector field:

curl F = (∂Fz/∂y - ∂Fy/∂z, ∂Fx/∂z - ∂Fz/∂x, ∂Fy/∂x - ∂Fx/∂y)

Plugging in the component functions of F(x, y, z), we get:

curl F = (2ez cos(x), -5ex cos(y), 9ey cos(z))

(b) To find the divergence of the vector field F(x, y, z), we use the formula for the divergence of a vector field:

div F = ∂Fx/∂x + ∂Fy/∂y + ∂Fz/∂z

Plugging in the component functions of F(x, y, z), we get:

div F = 5e^x sin(y) + 9e^y sin(z) + 2e^z sin(x)
(a) To find the curl of the vector field F(x, y, z) = (5e^x sin(y), 9e^y sin(z), 2e^z sin(x)), we need to compute the cross product of the del operator (∇) and F:

curl F = ∇ x F

curl F = ( (∂/∂y)(2e^z sin(x)) - (∂/∂z)(9e^y sin(z)), (∂/∂z)(5e^x sin(y)) - (∂/∂x)(2e^z sin(x)), (∂/∂x)(9e^y sin(z)) - (∂/∂y)(5e^x sin(y)) )

After computing the partial derivatives, we get:

curl F = ( 0, 5e^x cos(y) - 2e^z cos(x), 9e^y cos(z) - 5e^x cos(y) )

(b) To find the divergence of the vector field F(x, y, z), we need to compute the dot product of the del operator (∇) and F:

div F = ∇ ⋅ F

div F = (∂/∂x)(5e^x sin(y)) + (∂/∂y)(9e^y sin(z)) + (∂/∂z)(2e^z sin(x))

After computing the partial derivatives, we get:

div F = 5e^x sin(y) + 9e^y sin(z) + 2e^z sin(x)

learn more about vector field here: brainly.com/question/16180391

#SPJ11

Pls answer this asap

Answers

Answer:

  (C)  Neither

Step-by-step explanation:

You want to know if the function values in the table represent an even function, and odd function, or neither.

Symmetry

An Even function is symmetrical about the y-axis:

  f(-x) = f(x)

An Odd function is symmetrical about the origin:

  f(-x) = -f(x)

Application

The attached graph of the given points shows the function has no symmetry at all.

The table represents neither an even nor odd function.

Answer:

Neither

Step-by-step explanation:

In an even function, f(x) = f(-x).

Look at x = 2 and x = -2.

f(2) = -4; f(-2) = 2

Since f(2) ≠ f(-2), the function is not even.

In an odd function, f(x) = -f(-x).

Look at f(2) and f(-2).

f(2) = -4; f(-2) = 2

Since f(2) ≠ -f(-2), the function is not odd.

Answer: C  Neither

What is the cost of 0. 7 kg of apples, if 1 kg of apples cost 89. 50

Answers

In the proportion,  the cost of 0.7kg apple is Rs.62.65 .

What is proportion?

A percentage is created when two ratios are equal to one another. We write proportions to construct equivalent ratios and to resolve unclear values. a comparison of two integers and their proportions. According to the law of proportion, two sets of given numbers are said to be directly proportional to one another if they grow or shrink in the same ratio.

Here cost of 1kg apples = 89.50

Then cost of 0.7 kg apple = x.

Now using proportion,

=> 1   = 89.50

   0.7=  x

=> x = [tex]\frac{89.50\times0.7}{1}[/tex]

=> x = 62.65.

Hence the cost of 0.7kg apple is Rs.62.65 .

To learn more about proportion refer the below link

https://brainly.com/question/13604758

#SPJ4

The area of the region under the curve of a function f(x)= ax+b on the interval [0,4] is 16 square units. (A,b) ≠

Answers

There are infinitely many solutions to this equation. For example, one possible solution is a = 2, b = 0. Another possible solution is a = 1, b = 2.

How to find the area?

To find the area, we need to use the definite integral formula to calculate the area under the curve:

∫[0,4] f(x) dx = ∫[0,4] (ax + b) dx = 1/2 * a * x² + b * x |[0,4]

Substituting the limits of integration, we get:

1/2 * a * 4² + b * 4 - (1/2 * a * 0² + b * 0) = 16

Simplifying, we get:

8a + 4b = 16

Dividing by 4, we get:

2a + b = 4

Since (a,b) ≠ (0,0), there are infinitely many solutions to this equation. For example, one possible solution is a = 2, b = 0. Another possible solution is a = 1, b = 2.

Learn more about area

brainly.com/question/30307509

#SPJ11

Answer Immeditely Please

Answers

Answer:

it's ither 1 2 or 4 I'm still thinking but I'm pretty sure it's one of those answers

Determine the intercepts of the line.

Do not round your answers.

5x − 9 = −8y − 3
Please help

Answers

The intercepts of the line are (6/5, 0) and (0, -3/4).

We have,

To find the x-intercept, we need to set y = 0 and solve for x:

5x - 9 = -3

5x = 6

x = 6/5

So the x-intercept is (6/5, 0).

To find the y-intercept, we need to set x = 0 and solve for y:

-9 = -8y - 3

8y = -6

y = -3/4

So the y-intercept is (0, -3/4).

Therefore,

The intercepts of the line are (6/5, 0) and (0, -3/4).

Learn more about the equation of a line here:

https://brainly.com/question/23087740

#SPJ1

Please Help. All I have been getting is bot answers, and I actually need help with this. Thanks.



1. Two neighbors are each hosting a party. The first neighbor orders 5 large pizzas, each with a diameter of 16 inches. The second neighbor orders 9 small pizzas, each with a diameter of 12 inches. In terms of area, which party has more pizza? Explain. Show all work.



2. A toy train set has a circular track piece. The inner radius of the piece is 6 cm. One sector of the track has an arc length of 33 cm on the inside and 55 cm on the outside. What is the width of the track?

Answers

1. The second party has more pizza in terms of area

2. The width of the track is approximately 3.50 cm.

How to find which party has more pizza?

1. To compare the amount of pizza between the two parties, we need to find the total area of each set of pizzas. We can use the formula for the area of a circle, A = πr², where r is the radius of the circle.

For the large pizzas, the diameter is 16 inches, so the radius is 8 inches. The area of one pizza is:

A = π(8)²

A = 64π square inches

Since there are 5 pizzas, the total area of pizza for the first party is:

Total area = 5(64π) = 320π square inches

For the small pizzas, the diameter is 12 inches, so the radius is 6 inches. The area of one pizza is:

A = π(6)²

A = 36π square inches

Since there are 9 pizzas, the total area of pizza for the second party is:

Total area = 9(36π) = 324π square inches

Therefore, the second party has more pizza in terms of area.

How to find width of the track?

2. We can start by finding the length of the arc of the sector. We know that the arc length on the inside is 33 cm, so the angle of the sector can be found using the formula:

angle = (arc length / radius)

angle = (33 / 6) radians

To find the width of the track, we need to subtract the length of the inner circle from the length of the outer circle, and then divide by the angle of the sector. Let's call the width of the track "x". Then we have:

55 cm - 33 cm = 2π(6 cm + x) - 2π(6 cm)

22 cm = 2πx

x = 11 / π cm

So the width of the track is approximately 3.50 cm.

Learn more about Pizza width

brainly.com/question/9552582

#SPJ11

What is the max/min of the quadratic equation in factored form: f(x) = 0. 5(x+3)(x-7)? Is it a maximum or a minimum? Show your workor explain your reasoning. ​

Answers

The quadratic equation in factored form: f(x) = 0. 5(x+3)(x-7) have a minimum point. The minimum value of the function is -11.

To find the maximum or minimum of the quadratic equation in factored form f(x) = 0.5(x+3)(x-7), we need to convert it to standard form by expanding the terms:

f(x) = 0.5(x² - 4x - 21)

f(x) = 0.5x² - 2x - 10.5

The coefficient of x² is positive, so the parabola opens upwards and we have a minimum point.

To find the x-coordinate of the minimum point, we can use the formula x = -b/2a, where a = 0.5 and b = -2:

x = -(-2)/2(0.5) = 2

So the minimum point is at x = 2. To find the y-coordinate, we can substitute x = 2 into the equation:

f(2) = 0.5(2)^2 - 2(2) - 10.5 = -11

Therefore, the minimum value of the function is -11.

Learn more about minimum point at https://brainly.com/question/29288315?

#SPJ11

A boy cycles 5km from his home to school and 8km from his home to the market. The chief's camp is closer to the boys home than the market but further than the school. Write a compound inequality to show the distance from the boys home to the chief's camp.

Answers

A compound inequality to show the distance from the boys home to the chief's camp is 5 < d < 8

How to explain the inequality

The distance from the boy's home to the school is 5km.

The distance from the boy's home to the market is 8km.

The chief's camp is closer to the boy's home than the market, but further than the school

The chief's camp is closer to the boy's home than the market, so the distance from the boy's home to the chief's camp is less than 8km.

Putting these together, we can write a compound inequality to show the possible distances d from the boy's home to the chief's camp:

distance from home to school < distance < home to maket

5 < d < 8

The inequality is 5 < d < 8.

Learn more about Inequalities on

https://brainly.com/question/24372553

#SPJ1

Jane completed 8 homework problems in class. The function p(m) relates the


time (in minutes) Jane spent on her homework at home to the total number


of problems she completed. The input is the number of minutes worked. The


output is the number of problems completed.


p(m)= m/5+8


Which equation represents the inverse function m(p), which uses problems


completed as the input and gives minutes worked as the output?

Answers

Answer:

Step-by-step explanation:

5p-40

Answer this question please

Answers

This question can be answered as follows:

-4 + 12 = 8

How to answer the question

To answer this question, we can begin by searching for the number whose addition to -4 will yield 8 and the answer is 12. To determine the answer, we can first begin by assigning the figure x to the unknown variable. Thus, we will have:

-4 + x = 8

We collect like terms:

x = 8 + 4

= 12

So, the number that when added to -4 will yield 8, is 12. So, to find a missing number, we assign it a variable and equate to the specified end result.

Learn more about addition and subtraction here:

https://brainly.com/question/25421984

#SPJ1

If a data set has many very large data values, then the mean wil be
the median
a larger than
b. the same as
c. equal to
d. smaller than

Answers

A data set has many very large data values, then the mean will be larger than the median. The correct option is a.

In a dataset with large values, the mean is influenced by outliers and extreme values, whereas the median is not. The median is the value that divides the data into two equal halves, while the mean is calculated by summing up all values and dividing by the total number of values.

Thus, in a skewed dataset with large values, the mean tends to be pulled towards the larger values, resulting in a larger mean than the median. This effect is more pronounced in datasets with a high variance. Therefore, option (a) is the correct answer.

To know more about mean, refer here:

https://brainly.com/question/31101410#

#SPJ11

Keshia is working on a math worksheet with 50 problems she has completed 20 problems in 25 mins if she continue at this pace what will be her total time for her compete worksheet

Answers

It will take Keshia 62.5 minutes to complete the entire worksheet at the same pace.

To find the amount of total time it will take Keshia to complete the entire worksheet at the same pace, we can use a proportion:

20 problems / 25 minutes = 50 problems / x minutes

To solve for x, we can cross-multiply and simplify:

20 problems * x minutes = 25 minutes * 50 problems

20x = 1250

x = 1250 / 20

x = 62.5

Therefore, it will take Keshia 62.5 minutes to complete the entire worksheet at the same pace.

To learn more about total time, click here:

https://brainly.com/question/30928238

#SPJ11

Esteban has `3` kittens. According to the vet, each kitten is born with blue eyes and there is a `50\%` chance of it changing color once they reach three months.






Esteban decides to run a simulation using `3` coins, where heads represent the eyes changing color.






The table shows the results of his simulation.






Estimate the probability that at least one of Esteban’s kittens will still have blue eyes at three months old

Answers

There is an estimated probability of 0.875 (or 87.5%) that at least one of Esteban's kittens will still have blue eyes at three months old.

Let's use the complement rule to estimate the probability that at least one kitten will still have blue eyes at three months old:

P(at least one kitten with blue eyes) = 1 - P(no kitten with blue eyes)

To estimate P(no kitten with blue eyes), we can use the results of Esteban's simulation. We can see from the table that all three coins came up tails (representing no color change) in row 8, so that's one outcome where no kitten has blue eyes. There are a total of 2^3 = 8 possible outcomes, so the estimated probability of no kitten having blue eyes is:

P(no kitten with blue eyes) = 1/8 = 0.125

Therefore, the estimated probability that at least one kitten will still have blue eyes at three months old is:

P(at least one kitten with blue eyes) = 1 - P(no kitten with blue eyes) = 1 - 0.125 = 0.875

So, there is an estimated probability of 0.875 (or 87.5%) that at least one of Esteban's kittens will still have blue eyes at three months old.

To learn more about probability visit: https://brainly.com/question/30034780

#SPJ11

Subtract 1/4- 5/14 = type an integer or fraction

Answers

Answer:

-3/28

Step-by-step explanation:

In order to subtract fractions you first have to get a common denominator. The common denominator between 4 and 14 is 28 since 4(7) = 28 and 14(2) = 28. Then multiply the top and bottom of (1/4) by (7/7) and (5/14) by (2/2). This gives a common denominator and since you multiply by a fraction equal to 1, it doesn't change the value. Therefore, we get 7/28 - 10/28 = -3/28.

Researchers pose a question can pant sizes be predicted from a man's height? A random sample of 20 males and their pant size versus height

Answers

Answer:

Yes, there is evidence at the 10% significance level, but not at the 5% level

Step-by-step explanation:

Look at the p-value in bottom left corner (in the Height row).

It is less than 0.1, but greater than 0.05. Thus, Yes, there is evidence at the 10% significance level, but not at the 5% level. Got it right.

El mastil de un velero se halla unido a la proa y a la popa por dos cables que forman con la cubierta, angulo de 45 grados y 60 grados, respectivamente. si el barco tiene una longitud de 25m, ¿cual es la altura del mastil?

Answers

La altura del mástil es de aproximadamente 20.87 metros.

How to find mast height?

Para resolver el problema, se puede utilizar la ley de cosenos para encontrar la longitud del mástil:

c² = a² + b² - 2ab cos C

Donde:

c es la longitud del mástil (lo que se busca)

a es la longitud de la parte delantera del barco (25m)

b es la longitud de la parte trasera del barco (también 25m)

C es el ángulo entre a y b, que se puede calcular utilizando la tangente: tan C = 1.5 (pues tan 60° = √3, y tan 45° = 1)

Resolviendo para c:

c² = 25² + 25² - 2(25)(25)cos(arctan 1.5)

c = √(25² + 25² - 2(25)(25)(1/√(1+(1.5)²)))

c ≈ 31.08 m

Por lo tanto, la altura del mástil es de aproximadamente 31.08 metros.

Learn more about Para

brainly.com/question/23402398

#SPJ11

how does the area of a rectangle help you find the area of a triangle, rhombus, and parallelogram???

Answers

The area of a rectangle can be used to find triangle, rhombus and parallelogram since the shapes are related to one another

How does the area of a rectangle help you find the area of a triangle, rhombus, and parallelogram?

The area of a rectangle can be used to determine the area of a triangle, rhombus and parallelogram because these figures are somewhat similar to a rectangle.

When we join two triangles together at opposite end or we cut transversally between a rectangle from the diagonal, we would have two triangle.

When a rhombus is split across it's diagonal, it will produce two congruent triangles.

When a parallelogram is divided across it's diagonals, it produces two congruent triangles.

Learn more on area of a rectangle here;

https://brainly.com/question/2607596

#SPJ1




Fred goes to Old Navy and buys two pairs of jeans for $17. 99 each, three shirts at $5. 99 each, and a pack of socks for $3. He has a coupon for 25% off his entire purchase. Sales tax is 6. 5%. What is the total cost after the discount and tax?


A: $43. 07


B: $39. 95


C: $45. 49


D: $60. 65

Answers

The total cost after discount and tax is $46.28.

To find the total cost after the discount and tax, we need to first find the subtotal before tax.

Fred bought two pairs of jeans for $17.99 each, so the cost of the jeans is $17.99 x 2 = $35.98.

He also bought three shirts at $5.99 each, so the cost of the shirts is $5.99 x 3 = $17.97.

The pack of socks costs $3.

The subtotal before discount is $35.98 + $17.97 + $3 = $57.95.

With the 25% off coupon, Fred gets a discount of $57.95 x 0.25 = $14.49.

The new subtotal after discount is $57.95 - $14.49 = $43.46.

Finally, we need to add the 6.5% sales tax.

The sales tax is $43.46 x 0.065 = $2.82.

The total cost after discount and tax is $43.46 + $2.82 = $46.28.

Therefore, the closest answer is C: $45.49.

Learn more about Profit and Loss: https://brainly.com/question/9281343

#SPJ11

a repair shop charges per hour the amount of money they charge is below what equation does the repair shop use.
1 $70 Here are the answers i can chose from

c= 30h+ 40 C= 40h+30 C=30h+70 C= 40h
h is for hours and c is for charge
2 $100

3 $130

4 $160

5 $190

6 $220

Answers

The linear function for the repair cost is given as follows:

C(h) = 30h + 40.

How to define a linear function?

The slope-intercept representation of a linear function is given by the equation shown as follows:

y = mx + b

The coefficients m and b have the meaning presented as follows:

m is the slope of the function, representing the increase/decrease in the output variable y when the input variable x is increased by one.b is the y-intercept of the function, representing the numeric value of the function when the input variable x has a value of 0. On a graph, it is the value of y when the graph of the function crosses or touches the y-axis.

Each hour the cost increases by $30, hence the slope m is given as follows:

m = 30.

Hence the function is:

C(h) = 30h + b

When h = 2, C = 100, hence the intercept b is given as follows:

60 + b = 100

b = 40.

Then:

C(h) = 30h + 40.

More can be learned about linear functions at https://brainly.com/question/15602982

#SPJ1

y = 49 - 4x y = 73 - 7x

Answers

The solution of the system of equation

x = 8 and y = 17

How to solve system of equation?

System of equation can be solved using different method such as elimination method, substitution method and graphical method. Let's solve the system of equation by substitution method.

Therefore,

y = 49 - 4x

y = 73 - 7x

Hence, using substitution,

73 - 7x = 49 - 4x

73 - 49 = -4x + 7x

24 = 3x

divide both sides of the equation by 3

x = 24 / 3

x = 8

Therefore,

y = 73 - 7(8)

y = 73 - 56

y = 17

learn more on system of equation here: https://brainly.com/question/13737751

#SPJ1

Fill in the boxes 3(n+7) = (3)( ) + (3)( ) = +

Answers

the final result will be 3n + 21

what is ditribustion propety?

The Distributive Property is a mathematical property that states that when a single term is multiplied by a sum or difference, the result can be expressed as the sum or difference of the products of the term multiplied by each term inside the parentheses. It is often used to simplify expressions and equations in algebra. 3(x + 2) = 3x + 32 (distributing the 3 over the parentheses) = 3x + 6 (simplifying the products) This property is very useful when dealing with expressions containing variables or unknowns, as it allows us to simplify complex expressions and solve equations more easily.

using distribution propety

3(n + 7)

= (3)(n) + (3)(7)

= 3n + 21

Hence the final result will be 3n + 21

Learn more about ditribustion propety, by the following link

https://brainly.com/question/2807928

#SPJ4

The answer to this math problem please.

Answers

Answer:

C) 6

Step-by-step explanation:

n=6

6( 6 + 1) + 3 =45

36 + 6 + 3 = 45

36 + 9 = 45

A cross section of the cylinder with the cone removed is a ring. To find the area of the ring, find the area of the outer circle and of the inner circle. Then subtract the area of the inner circle from the outer circle.​

Answers

The area of the outer circle and of the inner circle is :  πr² and,  πx².

Then subtract the area of the inner circle from the outer circle is :

π (r - x) (r+x)

Here, we have,

Given:

Let the radius of outer circle i.e CA be r

Let the radius of inner circle i.e CB be x

The diagram is given below as attachment.

We know that,

area of circle = πR²

Then subtract the area of the inner circle from the outer circle is:

The area of the outer circle - area of the inner circle

= πr² -  πx²

= π (r² - x²)

=π (r - x) (r+x)

So, we get

Then subtract the area of the inner circle from the outer circle is:

π (r - x) (r+x)

Hence, The area of the outer circle and of the inner circle is :  πr² and,  πx².

Then subtract the area of the inner circle from the outer circle is :

π (r - x) (r+x)

Learn more about circle here:

brainly.com/question/11833983

#SPJ1

The function h is given by h(x)=log_2(x^2 -6). For what positive value of x does h(x)=4?

Answers

The function h is given by h(x)=log2(x² -6). The positive value of x that makes h(x) equal to 4 is approximately 4.69

We have the function:

h(x) = log2(x² - 6)

We want to find the value of x that makes h(x) equal to 4:

h(x) = 4

log2(x² - 6) = 4

We can rewrite this equation as:

2⁴ = x² - 6

16 = x² - 6

x²= 22

x = √22 (because we are looking for a positive value of x)

Therefore, the positive value of x that makes h(x) equal to 4 is approximately 4.69 (rounded to two decimal places).

Learn more about logarithm function at https://brainly.com/question/4563843

#SPJ11

Given that 3 is a primitive root modulo 25; Find a primitive root modulo 250

Answers

The primitive root modulo 250 is 103, as 3 is a primitive root modulo 25 we tested if it is also a primitive root modulo 250.

To find a primitive root modulo 250, we need to first factor 250 as 2 x 5³. Since 3 is a primitive root modulo 25, we can test if it is also a primitive root modulo 250.

Using Euler's totient function, we know that [tex]\phi(250)[/tex] = 100. Therefore, we only need to check if [tex]3^{20[/tex] (which is [tex]3^{\phi(250)/2[/tex]) is congruent to -1 modulo 250.

Calculating [tex]3^{20[/tex] modulo 250 gives us 1. Since [tex]3^{20[/tex] is not congruent to -1 modulo 250, 3 is not a primitive root modulo 250.

To find a primitive root modulo 250, we can use a common method called the "index cycling" method. We can start with a primitive root modulo 5³ = 125 and then test the other primitive roots modulo 2³ = 8 until we find a primitive root modulo 250.

Using a computer or calculator, we can find that 2 is a primitive root modulo 125. To find a primitive root modulo 250, we can test the numbers 2, 2 + 125, 2 + 2125, and 2 + 3125 until we find a primitive root.

Testing these numbers, we find that 2 + 3*125 = 377 is a primitive root modulo 250. Therefore, 377 is a primitive root modulo 250.

Learn more about the primitive root modulo at

https://brainly.com/question/14766413

#SPJ4

Find the properties for the ellipse with the equation x^2/169 + y^2/144 = 1

latus rectum =

a. 288/13
b. 328/12
c. 288/12

Answers

The latus rectum of an ellipse is 288/13 when the ellipse equation is given as [tex]x^2/169 + y^2/144 = 1[/tex]. Option A is correct.

The standard form of an ellipse equation is given as  

[tex](x2/a2) + (y2/b2) = 1[/tex]

where :

a = lengths of the semi-major axes

b = length of semi-minor axes

The length of the chord through one of the foci that are perpendicular to the major axis is defined as the latus rectum of an ellipse.

From the given data the equation of the ellipse is given as :

x² / 169 + y² / 144 = 1

By comparing the standard equation and the given equation of the ellipse we get :

a² = 169

a = √169

[tex]a = 13[/tex]

b² = 144

b = √144

[tex]b = 12[/tex]

The distance between the center and one of the foci is given by

c = √(a² - b²)

= √(13² - 12²)

= 5

We can find the latus rectum by substuting a and b values in the latus rectum of an ellipse formula,

= 2×b²/a

= 2 × 12² / 13

= 288/13

Therefore, the latus rectum of an ellipse is 288/13.

To learn more about the latus rectum of an ellipse :

https://brainly.com/question/30017551

#SPJ4

A teacher asked three different students to write the conditions that would result in a triangle. Which of the following students listed conditions that would result in more than one triangle?

Answers

The condition that will result in more than one triangle is C. Student III.

How the conditions will result in more than one triangle

The conditions listed by the third student will result in more than one triangle because we are given all three angles. As a rule in math, some conditions will determine if there is more than one triangle. One of them is this:

Rule 1:

If all three angles of the triangle are given and they all add up to exactly 180°, it is possible to get more than one triangle.  In the third option, we are given angles 62°, 36°, and 82°, so different triangles can be constructed. Also, they all add up to give 180°. The condition is satisfied.

Rule 2:

Also, as a rule, if we have two angles that do not add up to 180° and one side, then only one unique triangle can be obtained. This is the case for student A who is given angles A and B and a side length of 5cm. (ASA)

Rule 3:

Student 2 will also produce a unique triangle because there are three sides that meet the triangle inequality theorem. (SSS)

Learn more about triangles here:

https://brainly.com/question/1058720

#SPJ1

Review Questions

1. A washer and a dryer cost $1004 combined. The washer costs $54 more than the dryer. What is the cost of the dryer?

Please use your work.

Answers

The calculated cost of the dryer is $475.

What is the cost of the dryer?

Let's assume that the cost of the dryer is x dollars.

According to the problem, the cost of the washer is $54 more than the dryer.

Therefore, the cost of the washer is (x + $54).

We are given that the combined cost of the washer and the dryer is $1004. So we can set up the equation:

x + (x + $54) = $1004

Simplifying the equation:

2x + $54 = $1004

2x = $950

x = $475

Therefore, the cost of the dryer is $475.

Read more about expression at

https://brainly.com/question/15775046

#SPJ1

Jay has five fewer $100 bills in his wallet than $50 bills. If he has only $50 and $100 bills in his wallet, and the total amount of money in his wallet is $1300, let’s find the number of notes of each denomination.


Let’s assume the number of $50 bills as x. So, in terms of x, the number of $100 notes can be represented as ____.



Now, the total amount contributed by $50 notes is $50x



And. The total amount contributed by $100 bills is $100×( ____ )

Answers

Jay has 7 $100 bills in his wallet.

Let's assume the number of $50 bills as x. So, in terms of x, the number of $100 notes can be represented as x - 5.

Now, the total amount contributed by $50 notes is $50x.

And the total amount contributed by $100 bills is $100*(x-5).

Since the total amount of money in his wallet is $1300, we can write an equation:

$50x + $100*(x-5) = $1300

Simplifying this equation, we get:

$50x + $100x - $500 = $1300

$150x = $1800

x = 12

So, Jay has 12 $50 bills in his wallet.

Using the earlier equation, the number of $100 bills can be found:

x - 5 = 12 - 5 = 7

Therefore, Jay has 7 $100 bills in his wallet.

To learn more about equation, refer below:

https://brainly.com/question/29657983

#SPJ11

Other Questions
"America has a problem...it is that of the Which of the lines below match the function given? y = x + 5 A graph with 4 lines, labeled A, B , C, and D. Line A contains the points 0 comma 0, and 2 comma 2. Line B contains the points 0 comma 5, and 2 comma 7. Line C contains the points 0 comma 0, and 2 comma 4. Line D contains the points 0 comma 2, and 1 comma 5. What is the value of the x-coordinate of point A?a) sin (pi/6)b) cos (pi/6)c) sin (pi/3)d) cos (pi/3)e) sin (2pi/3)f) cos (2pi/3) A major car manufacturer wants to test a new engine to determine if it meets new air pollution standards. Safety regulations require that the mean emission of all engines of this type must be no greater than 20 parts per million (ppm) of carbon. If it is higher than that, they will have to redesign parts of the engine. A random sample of 25 engines is tested and the emission level of each is determined. The sample mean is calculated to be 20. 4 ppm and the sample standard deviation is calculated to be 2. 0 ppm. It is known that emission levels are normally distributed with standard deviation 1. 6 ppm. We would like to test whether the true mean emission level for the new engine is greater than 20 ppm. The test statistic for the appropriate test of significance is: ________ lease help!!! I need this urgently. I will mark you brainliest. Write a speech of 3 minutes on the topic : is climate change really an issue. In you speech you should identify you sources, be able to discuss the general thread and bias of the article /research, and quote from the article /research Why does greg usually bring his lunch? Lunch Money book can someone pls answer this ASAPchoices: Union forces were able to use the blockade to gain international support.Confederate states were unable to use agricultural exports as a source of revenue.Union forces were able to earn money for supplies to selling confiscated bales of cotton.AND WHAT ERA WAS THIS Question 7!!Total area = area of parallelogram + area of trapezoid Helpp me please i need to finish before flvs logs me out helpppppquestion 2 (essay worth 10 points)(05.05 mc)a food truck did a daily survey of customers to find their food preferences. the data is partially entered in the frequency table. complete the table to analyze the data and answer the questions:part a what percentage of the survey respondents do not like both hamburgers and burritos? (2 points)part b: what is the marginal relative frequency of all customers that like hamburgers? (3 points)part c: use the conditional relative frequencies to determine which data point has strongest association of its two factors. use complete sentences to explain your answer. (5 points) An insulated collar is made to cover a pipe. Find the volume of the material used to make the collar. Let r = 3 inches, R = 5 inches, and h = 21 inches. Use 3. 14 for p, and round to the nearest hundredth What happens in a decomposition reaction? A. Two ions trade places. B. Two substances combine to form one substance. C. The charges of the atoms change. D. Compounds break down into smaller compounds. I have a $500 budget for an event what would be good food options that will be around the $500 mark if I have 50 guests attending? cual es el grado de polinomios de -8^3 xy^4 por favor me ayudan doy corona Your parents decide that they will help you out the first two months you are at collegeby helping you buy groceries for your meals, but they won't cover the cost of you goingout to eat at restaurants or fast food.for the first month you submit your receipts, 13 are for fast food and four for kroger, andtotaled $487. the second month you submit receipts for six fast food meals and two forkroger, and totaled $232. the receipts do not list the per-item price.a. write the two equations for the cost of buying groceries and meals. (4 points) 13f + 4k = 487 65+2k = 232ib. what were the average costs of one fast food meal and of one trip to kroger?show your work and justify your thinking. (6 points) Reread the line "Let the knives be sharp and clean." What image does this evoke? How might you correctly use a relative clause to combine sentences 3 and 4?A Willa Cather wrote during the late 1800s and early 1900s to contribute to that American tradition. B Willa Cather contributed to the American Tradition of the late 1800s and early 1900s. C Willa Cather, who wrote during the late 1800s and early 1900s, contributed to that American tradition. D Writing during the late 1800s and early 1900s, Willa Cather contributed to that American tradition A company recorded 2 days of accrued salaries of $1,450 for its employees on january 31. on february 9, it paid its employees $7,100 for these accrued salaries and for other salaries earned through february 9. assuming the company does not prepare reversing entries, the january 31 and february 9 journal entries are: The maximum walking speed S, in feet per second, of and animal can be modeled by the equation S= square root of gL,where g+32ft/sec and L is the lenght, in feet, of the animal's leg. To the nearest hundredth,how ,amy times greater is the maximum walking speed of a giraffe with a leg of 6 feet than a hippopota,us with a leg of 3 feet? The table shows the cost of various expenses Rosaria expects to pay for one year at a private four-year college while living on campus.CategoryAmounttuition$36,259room/board$12,258books/lab fees$1,988How much should Rosaria expect to pay? A lake currently has a depth of 30 meters. As sediment builds up in the lake, its depth decreases by 2% per year. This situation represents:A. Exponential decayB. Exponential growthThe rate of growth or decay, r, is equal to: A. 1. 02B. 0. 02C. 0. 98So the depth of the lake each year is ______ times the depth in the previous year. A. 0. 98B. 0. 02C. 1. 02It will take between _____ years for the depth of the lake to reach 26. 7 meters. A. 3 and 4B. 11 and 12C. 9 and 10D. 5 and 6